How to calculate this integral

  • Thread starter mark.laidlaw19
  • Start date
  • Tags
    Integral
In summary: I know, from working which uses the residue theorem, that this integral should equal 10∏/(27√3), and have been able to compute all other parts of the question.I tried partial fractions, but because the denominator is of degree 9, this would take an incredibly long time, longer than I believe is necessary.I have tried integration by parts, but this does not seem to work, as I do not get the answer that I know to be correct from using the residue theorem.Basically, what I would appreciate is a suggestion of another method, and perhaps a overview of the initial steps, to do an integral like this.Any help is greatly
  • #36
haha, yeah me too. I was really unsure about how your method would get back to the original problem. That is one of the lesser-used methods of calculating difficult integrals, for sure! I've probably only used a method like that once or twice.
 
Physics news on Phys.org
  • #37
BruceW said:
haha, yeah me too. I was really unsure about how your method would get back to the original problem. That is one of the lesser-used methods of calculating difficult integrals, for sure! I've probably only used a method like that once or twice.

Thanks Bruce! Yes, you are right, this is one of the lesser-used methods but I recently came to know how powerful it is in evaluating some of the most awkward and difficult integrals. :)

Oh and BTW, I came up with a faster method to evaluate the definite integral.

In the definite integral, ##\int_0^{\infty} \frac{dx}{x^3+a^3}##, use the substitution ##x^3=a^3t## to obtain:
$$\frac{1}{3a^2}\int_0^{\infty} \frac{t^{-2/3}}{1+t}\,dt$$
Then I use the result:
$$\int_0^{\infty} \frac{t^{-b}}{1+t}\,dt=\Gamma(b)\Gamma(1-b)=\frac{\pi}{\sin(\pi b)}$$
Hence,
$$\frac{1}{3a^2}\int_0^{\infty} \frac{t^{-2/3}}{1+t}\,dt=\frac{2\pi}{3\sqrt{3}a^2}$$

@Mark: The result I used is easy enough to prove once you know the gamma function, I suppose you are going to start with it soon. :)
 
Last edited:
  • #38
Pranav-Arora said:
I am sorry but I will have to disagree here (maybe its a matter of choice). Your suggestion would give nasty cuberoots and the subsequent steps would be much harder to deal with than they already are. Using ##a^3## gives a nicer expression.

For the indefinite integral, you are absolutely correct. However, for the definite integral it does not make much difference; in fact, IMHO the ##x^3+a## form really is a bit easier than the ##x^3+a^3## form, inasmuch as there are fewer opportunities for errors, etc.
 
  • #39
Pranav-Arora said:
Once the definite integral is evaluated, you will get something like
$$\int_0^{\infty} \frac{dx}{a^3+x^3}=f(a)$$
Since you need ##(a^3+x^3)^3## in the denominator, you simply differentiate both the sides with respect to ##a## twice and then substitute ##a=1##.

And ##f(a)## comes out to be a very nice expression. :)

Thank you very much. I see how this will get the correct denominator, but when you twice differentiate [tex]\int_0^R{\frac{1}{a^3+x^3}}[/tex]wouldn't you end up with [tex]\int_0^R \frac{12a^4-6ax^3}{(a^3+x^3)^3}\,dx[/tex] which will still have an x in the numerator even after setting a=1?
 
  • #40
mark.laidlaw19 said:
Thank you very much. I see how this will get the correct denominator, but when you twice differentiate [tex]\int_0^R{\frac{1}{a^3+x^3}}[/tex]wouldn't you end up with [tex]\int_0^R \frac{12a^4-6ax^3}{(a^3+x^3)^3}\,dx[/tex] which will still have an x in the numerator even after setting a=1?

Your second ##a##-derivative is
[tex] \frac{\partial^2}{\partial a^2} \frac{1}{x^3+a^3}
= \frac{18 a^4}{(x^3+a^3)^3} - \frac{6a}{(x^3+a^3)^2}[/tex]
and you get the integral of the second term from
[tex] \frac{\partial}{\partial a} \frac{1}{x^3+a^3} = -\frac{3a^2}{(x^3+a^3)^2}[/tex]

However, as I said before, if you look at the definite integral you can just use
[tex] \frac{\partial^2}{\partial b^2} \frac{1}{x^3+b} = \frac{2}{(x^3+b)^3}[/tex]
in the integral
[tex] \int_0^{\infty} \frac{dx}{x^3+b} = \frac{2 \pi \sqrt{3}}{9\, b^{2/3}}[/tex]
 
  • #41
Is there some reason you didn't evaluate this integral using contour integration?
 
  • #42
vela said:
Is there some reason you didn't evaluate this integral using contour integration?

questions asked explicitly for parametrization. Otherwise it would be much easier i agree
 
  • #43
mark.laidlaw19 said:
Thank you very much. I see how this will get the correct denominator, but when you twice differentiate [tex]\int_0^R{\frac{1}{a^3+x^3}}[/tex]wouldn't you end up with [tex]\int_0^R \frac{12a^4-6ax^3}{(a^3+x^3)^3}\,dx[/tex] which will still have an x in the numerator even after setting a=1?

No. You can simplify a bit after the first derivative. :)

You have:
$$\int_0^{\infty} \frac{dx}{a^3+x^3}=\frac{2\pi}{3\sqrt{3}a^2}$$
Differentiating once both the sides give:
$$\int_0^{\infty} \frac{3a^2}{(a^3+x^3)^2}\,dx=\frac{4\pi}{3\sqrt{3}a^3} \Rightarrow \int_0^{\infty}\frac{dx}{(a^3+x^3)^2}= \frac{4\pi}{9\sqrt{3}a^5}$$
Differentiate again:
$$\int_0^{\infty} \frac{6a^2}{(a^3+x^3)^3}\,dx=\frac{20\pi}{9\sqrt{3}a^6} \Rightarrow \int_0^{\infty} \frac{dx}{(a^3+x^3)^2}=\frac{10\pi}{27\sqrt{3}a^8}$$

Now just substitute ##a=1##. ;)
 

Similar threads

  • Calculus and Beyond Homework Help
Replies
3
Views
225
  • Calculus and Beyond Homework Help
Replies
7
Views
686
  • Calculus and Beyond Homework Help
Replies
5
Views
781
  • Calculus and Beyond Homework Help
Replies
11
Views
1K
  • Calculus and Beyond Homework Help
Replies
9
Views
939
  • Calculus and Beyond Homework Help
Replies
7
Views
958
  • Calculus and Beyond Homework Help
Replies
9
Views
738
  • Calculus and Beyond Homework Help
Replies
10
Views
1K
Replies
25
Views
1K
  • Calculus and Beyond Homework Help
Replies
16
Views
1K
Back
Top